Đến nội dung

Secrets In Inequalities VP nội dung

Có 298 mục bởi Secrets In Inequalities VP (Tìm giới hạn từ 05-06-2020)



Sắp theo                Sắp xếp  

#385429 { pi + i | 1 ≤ i ≤ n} và { pi - i | 1 ≤ i ≤ n} tạo thành...

Đã gửi bởi Secrets In Inequalities VP on 10-01-2013 - 21:51 trong Số học

Tìm tất cả các số nguyên dương n sao cho tồn tại một hoán vị (p1, p2,..., pn) của các số (1, 2, ..., n) thỏa mãn điều kiện các tập hợp { pi + i | 1 ≤ i ≤ n} và { pi - i | 1 ≤ i ≤ n} tạo thành hệ thặng dư đầy đủ modulo n

Do { pi + i | 1 ≤ i ≤ n} tạo thành hệ thặng dư đầy đủ (HĐĐ) modulo $n$ nên : $\sum_{i=1}^{n}{p_i}+i\equiv \sum_{i=1}^{n}i(Modn)$
Mà do (p1, p2,..., pn) là hoán vị của tập (1, 2, ..., n) nên $\sum_{i=1}^{n}{p_i}+i\equiv 2\sum_{i=1}^{n}i(Modn)$
suy ra $\sum_{i=1}^{n}i\equiv 2\sum_{i=1}^{n}i(Modn)\Rightarrow \sum_{i=1}^{n}i\equiv 0(Modn)\Rightarrow 1+2+...+n\equiv 0(Modn)\Rightarrow \frac{n(n+1)}{2}\equiv 0(Modn)$
Suy ra $n$ phải lẻ .
Do { pi + i | 1 ≤ i ≤ n} cũng tạo thành HĐĐ modulo $n$ nên :$2\sum_{i=1}^{n}i^{2}\equiv \sum_{i=1}^{n}(({p_i}+i)^2+({p_i}-i)^2)\equiv \sum_{i=1}^{n}2{p_i}^{2}+2i^{2}$
Mà do (p1, p2,..., pn) là hoán vị của tập (1, 2, ..., n) nên $\sum_{i=1}^{n}2{p_i}^{2}+2i^{2}\equiv 4\sum_{i=1}^{n}i^{2}(Modn)$
Suy ra $2\sum_{i=1}^{n}i^{2}\equiv 4\sum_{i=1}^{n}i^{2}(Modn)\Rightarrow 2\sum_{i=1}^{n}i^{2}\equiv0(Modn)\Rightarrow 2.(1^{2}+2^{2}+...+n^{2})\equiv 0(Modn)\Rightarrow \frac{n(n+1)(2n+1)}{3}\equiv 0(Modn)$
Suy ra $n$ không chia hết cho 3.
Từ $2$ điều trên ta có : $(n,6)= 1$
Bây giờ ta chứng minh là nếu $(n,6)= 1$ thì tồn tại một hoán vị (p1, p2,..., pn) thỏa mãn bài toán.
Chọn ${p_i}\equiv 2i(Modn)$, ${p_i}\in {1, 2, ..., n}$. Ta có {p1, p2,..., pn} thỏa đề vì $pi + i | 1\leq i\leqn\equiv 3i | 1 \leq i \leq n$ và $pi - i | 1\leq i\leq n \equiv i | 1 \leq i \leq n$ là các hệ thặng dư đầy đủ mô-đun n.



#381566 Chứng minh tồn tại $k_1,k_2,...,k_s$ để $b_1+m_1k_1=b_2+m_2k_2...

Đã gửi bởi Secrets In Inequalities VP on 29-12-2012 - 14:37 trong Số học

Cho $m_1,m_2,...,m_s$ là các số tự nhiên thỏa $(m_i,m_j)=1$ với mọi $i \ne j$. Cho $b_1,b_2,...,b_s \in \mathbb{Z}$. Chứng minh tồn tại $k_1,k_2,...,k_s \in \mathbb{Z}$ sao cho:
$$b_1+m_1k_1=b_2+m_2k_2=...=b_s+m_sk_s$$

Xét hệ phương trình đồng dư : $x\equiv {b_i} (Mod{m_i})$
Theo định lí thặng dư của Tàu thì hệ luôn có nghiệm $c$.$\Rightarrow c= {b_i}+{m_i}{k_i}$ suy ra $Q.E.D$



#381559 Chứng minh $m\vdots 1979$

Đã gửi bởi Secrets In Inequalities VP on 29-12-2012 - 14:11 trong Số học

Cho m,n là các số nguyên dương sao cho:
$\frac{m}{n}=1-\frac{1}{2}+\frac{1}{3}-\frac{1}{4}+...-\frac{1}{1318}+\frac{1}{1319}$
Chứng minh $m\vdots 1979$

Ta thấy $1979$ là số nguyên tố .
Ta có : $\frac{m}{n}=1-\frac{1}{2}+\frac{1}{3}-\frac{1}{4}+...-\frac{1}{1318}+\frac{1}{1319}$
$$=(1+\frac{1}{2}+\frac{1}{3}+\frac{1}{4}+...+\frac{1}{1318}+\frac{1}{1319})-2.(\frac{1}{2}+\frac{1}{4}+...+\frac{1}{1318})$$
$$=(1+\frac{1}{2}+\frac{1}{3}+\frac{1}{4}+...+\frac{1}{1318}+\frac{1}{1319})-(1+\frac{1}{2}+...+\frac{1}{659})$$
$$=\frac{1}{660}+\frac{1}{661}...+\frac{1}{1318}+\frac{1}{1319}$$
$$=(\frac{1}{660}+\frac{1}{1319})+(\frac{1}{661}+\frac{1}{1338})+...$$
$$=\frac{1979}{660.1319}+\frac{1979}{661.1318}+...$$
$$=1979.(\frac{1}{660.1319}+\frac{1}{661.1318}+...)= 1979.\frac{p}{q}$$
trong đó $q=660.661...1319\Rightarrow (q,1979)=1$
Ta có : $\frac{m}{n}= 1979.\frac{p}{q}\Rightarrow mq=1979np\Rightarrow mq\vdots 1979$
Mà $(q,1979)=1\Rightarrow m\vdots 1979\Rightarrow Q.E.D$



#381558 Với mỗi số nguyên dương n, ký hiệu S(n) là tổng tất cả các chữ số trong biểu...

Đã gửi bởi Secrets In Inequalities VP on 29-12-2012 - 13:55 trong Số học

Với mỗi số nguyên dương n, ký hiệu S(n) là tổng tất cả các chữ số trong biểu diễn thập phân của n.
Xét các số nguyên dương m là bội của 2003. Hãy tìm giá trị nhỏ nhất của S(m).

Đặt $p=2003\in\mathbb{P}$.
+ Nếu $S(n)= 1\Rightarrow n= 100...000= 10^{^{k}}\not\vdots 2003\Rightarrow S(n)> 1$
+ Nếu $S(n)= 2$$\Rightarrow n= 200...000$ hoặc $n=100...100...00= 10^{i}+10^{j}$
Ta có $n=200...000= 2.10^{k} \not\vdots 2003$ $\Rightarrow n=10^i+10^j$
$\Rightarrow 10^i+10^j\vdots 2003=p\Rightarrow 10^i= -10^j(Modp)$
$\Rightarrow 10^{i-j}= -1(Modp)$$\Rightarrow 10^k= -1(Modp)$ ( $k=i-j$ )
Dễ thấy $2^{10}= 1024\equiv 10^7(Modp)$ , suy ra :
$(2^{5k})^{2}= 2^{10k}\equiv (10^{k})^{7}\equiv -1(Modp)$
$\Rightarrow -1$ là số chính phương mod p suy ra $p$ có dạng $4k+1$
$\Rightarrow 2003=p= 4k+1\Rightarrow k= \frac{2002}{4} \not \in \mathbb{Z}$
Suy ra vô lí $\Rightarrow S(n)> 2$
+ Bây giờ ta chứng minh là có $n$ để $S(n)= 3$
Vì $p=2003\neq 8k-1\neq 8k+1$ suy ra $2$ không là SCP mod p $\Rightarrow 2^{\frac{p-1}{2}}\equiv -1(Modp))$
Mà $10^{7}\equiv 2^{10}(Modp)\Rightarrow 2.10^{700}\equiv 2^{1001}= 2^{\frac{p-1}{2}}\equiv -1 (Modp)$
$\Rightarrow (2.10^{700}+1)\vdots p$ .Mà $S(2.10^{700}+1)= 3$ nên khẳng định của ta là đúng.
Vậy $minS(n)= 3$.Xảy ra khi chẳng hạn $n= 2.10^{700}+1$



#385450 \[ a^3+b^3+c^3+6\ge (a+b+c)^2 \]

Đã gửi bởi Secrets In Inequalities VP on 10-01-2013 - 22:27 trong Bất đẳng thức và cực trị

$ a,b,c>0:\; abc=1 $ CMR
\[ a^3+b^3+c^3+6\ge (a+b+c)^2 \]

Lâu lâu ghé box BĐT chém tí :)) !
Theo BĐT $Schur$ ta có :
$$a^{3}+b^{3}+c^{3}+3abc\geq ab(a+b)+bc(b+c)+ca(c+a)$$
$$\Rightarrow 3(a^{3}+b^{3}+c^{3})+9abc\geq 3(ab(a+b)+bc(b+c)+ca(c+a))$$
$$\Rightarrow 4(a^{3}+b^{3}+c^{3})+15abc\geq a^{3}+b^{3}+c^{3}+3(ab(a+b)+bc(b+c)+ca(c+a))+6abc$$
$$\Rightarrow 4(a^{3}+b^{3}+c^{3})+15abc\geq (a+b+c)^3$$
$$\Rightarrow 4(a^{3}+b^{3}+c^{3})+15\geq (a+b+c)^3$$
$$\Rightarrow 4(a^{3}+b^{3}+c^{3})+24\geq (a+b+c)^3+9$$
Theo $AM-GM$ thì :
$$\frac{(a+b+c)^3}{3}+\frac{(a+b+c)^3}{3}+ \frac{(a+b+c)^3}{3}+9\geq 4\sqrt[4]{\frac{(a+b+c)^9}{3}}$$
$$ 4\sqrt[4]{\frac{(a+b+c)^9}{3}}\geq 4\sqrt[4]{\frac{(a+b+c)^8.3\sqrt[3]{abc}}{3}}= 4(a+b+c)^2$$
Do đó : $4(a^{3}+b^{3}+c^{3})+24\geq 4(a+b+c)^2$
Chia $2$ vế cho $4$ ta được ngay $Q.E.D$



#393852 $a_{n}$ là số nguyên dương,$\forall n\in...

Đã gửi bởi Secrets In Inequalities VP on 06-02-2013 - 17:26 trong Dãy số - Giới hạn

Cho dãy số $(a_{n})$:$(a_{n})\left\{\begin{matrix} a_{0}=1\\ a_{n}=(7a_{n-1}+\sqrt{45a_{n-1}^{2}-36})\div 2,\forall n\in \mathbb{N}^{*} \end{matrix}\right.$
Chứng minh:
a)$a_{n}$ là số nguyên dương,$\forall n\in \mathbb{N}$
b)$a_{n+1}a_{n}-1$ là số chính phương,$\forall n\in \mathbb{N}$

a) Bằng qui nạp dễ cm $({a_n})$ là dãy tăng.
Từ hệ thức ban đầu ta suy ra : $$(2a_{n}-7a_{n-1})^{2}= 45a_{n-1}-36$$
Rút gọn ta được : $$2a_{n}^{2}-7a_{n}a_{n-1}+2a_{n-1}^{2}+18=0$$
Thay $n$ bởi $n+1$ :$$2a_{n+1}^{2}-7a_{n+1}a_{n}+2a_{n}^{2}+18=0$$
Trừ từng vế $$\Rightarrow 2a_{n+1}^{2}-2a_{n-1}^{2}+7a_{n}(a_{n-1}-a_{n+1})= 0$$
$$\Rightarrow (a_{n+1}-a_{n-1})(2a_{n+1}-7a_{n}+2a_{n-1})= 0$$
Vì $({a_n})$ là dãy tăng nên $a_{n+1}-a_{n-1}> 0\Rightarrow 2a_{n+1}-7a_{n}+2a_{n-1}=0$
Từ hệ thức này kết hợp qui nạp dễ thấy $Q.E.D$

b) Ta có : $a_{0}=1;a_{1}=5;a_{2}=34;2a_{n+1}-7a_{n}+2a_{n-1}=0$
Qui nạp một tí là ra : $a_{n+1}a_{n-1}-a_{n}^{2}=9$
Thêm bớt thành hằng đẳng thức tí nữa là ra .



#507979 $$P=(a^2+b^2-ab)(b^2+c^2-bc)(a^2+c^2-ac)$$

Đã gửi bởi Secrets In Inequalities VP on 20-06-2014 - 10:20 trong Bất đẳng thức và cực trị

Giả sử $a= min$

Suy ra $a^{2}+b^{2}-ab\leq b^{2};a^{2}+c^{2}-ac\leq c^{2}$

Do đó $P\leq b^2c^2(b^2-bc+c^2)= b^2c^2[(b+c)^2-3bc]\leq b^2c^2(3^2-3bc)$

Đến đây $AM-GM$ hoặc ksht là ra.




#414790 $x^2+y^2+z^2-wp=0$

Đã gửi bởi Secrets In Inequalities VP on 25-04-2013 - 20:13 trong Số học

 

CMR: với mọi số nguyên tố cho trước $p$ thì thì tồn tại số tự nhiên $x,y,z,w$ thỏa $x^2+y^2+z^2-wp=0$ và $0<w<p$

 

 

+Nếu $p=2$ chọn ngay $x=0,y=z=w=1$ ta có đpcm

+Nếu $p>2$.CHọn $z=1$.Xét các tập $A={x^2}$, $B={-y^2-1}$ với $x$ và $y$ lấy giá trị trong tập $C$ từ $0$ đến $\frac{p-1}{2}$.

Dễ thấy nếu $a,b\in C$ thì $a^{2}\not\equiv b^{2} (mod p)$ bởi vì nếu ngc lại thì $(a-b)(a+b)\vdots p$ nhg cả hai số này đều nhỏ hơn $p$ nên vô lí.

$\Rightarrow$ các phần tử trong $A$ và $B$ có số dư khác nhau khi chia cho $p$.

Mà $|A |+| B |= p+1> p$ nên tồn tại $2$ phần tử $x,y$ thuộc $A$ và $B$ sao cho $$x^{2}\equiv -y^{2}-1(Modp)\Rightarrow x^2+y^2+1= wp$$

$\Rightarrow 0< w=\frac{x^2+y^2+1}{p}\leq \frac{2(\frac{p-1}{2})^2+1}{p}< p$

Vậy chọn $x,y,z,w$ như trên ta có đpcm.




#414786 $P(x)=(x-a_1)^{2}(x-a_2)^{2}...(x-a_n)^{2}...

Đã gửi bởi Secrets In Inequalities VP on 25-04-2013 - 19:59 trong Đa thức

Chứng minh rằng với mọi số nguyên $a_1,a_2,...,a_n$ đôi một khác nhau,thì đa thức:$P(x)=(x-a_1)^{2}(x-a_2)^{2}...(x-a_n)^{2}+1$ không thể biểu diễn thành tích của hai đa thức (bậc dương) với hệ số nguyên.

Phản chứng giả sử $P(x)=g(x).h(x)$ ,$g(x),h(x) \in R[x]$ , $degg(x)+degh(x)= 2n$,$deg g(x)\leq deg h(x)\Rightarrow degg(x)\leq n$

$\Rightarrow g({a_i}).h({a_i})=P({a_i})= 1$$\forall i=1,2,...,2013$

$\Rightarrow$ $g(x)$ và $h(x)$ cùng đồng nhất bằng $1$ hoặc $-1$.

Nếu có $i,j$ sao cho $g({a_i})= 1; g({a_j})= -1\Rightarrow g({a_i}).g({a_j})< 0\Rightarrow \exists {x_0}:g({x_0})= 0$

$\Rightarrow P(x)$ có nghiệm ${x_0}$ vô lí vì $P(x)$ luôn dương với mọi $x$.

Do đó chỉ có thể xảy ra $g({a_i})=h({a_i})=1$ hoặc $g({a_i})=h({a_i})=-1$. với mọi $i$ chạy từ $1$ đến $n$

Xét TH $g({a_i})=h({a_i})=1$ , cái kia cmtt.

Vì $g({a_i})=1$ nên đa thức $g(x)-1$ có $n$ nghiệm từ ${a_1}$ đến ${a_n}$ nên bậc của nó sẽ lớn hơn hay bằng $n$ 

$degg(x)= degh(x)= n$

$\Rightarrow g(x)-1= c(x-{a_1})(x-{a_2})...(x-{a_n})$ , $ h(x)-1= d(x-{a_1})(x-{a_2})...(x-{a_n})$.

Mà $P(x)= g(x).h(x)$ nên so sánh hệ số cao nhất $2$ vế ta có $cd=1$.Giả sử là $c=d=1$.

Lại do $P(x)= g(x).h(x)$ $\Rightarrow \prod_{i=1}^{n}(x-{a_i})^{2}= (\prod_{i=1}^{n}(x-{a_i})+1).(\prod_{i=1}^{n}(x-{a_i})+1)$

$\Leftrightarrow 2\prod_{i=1}^{n}(x-{a_i})= 0\forall x\in R$.Vô lí.

Vậy giả sử sai và ta có đpcm




#410835 Chứng minh $d$ chia hết cho $2010$.

Đã gửi bởi Secrets In Inequalities VP on 06-04-2013 - 19:57 trong Số học

Cho hai đa thức hệ số nguyên $P(x)$ và $Q(x)$ thỏa mãn $P(x^3)+xQ(x^3)$ chia hết cho $x^2+x+1$. Gọi $d$ là ước chung lớn nhất của$P(2011)$ và $Q(2011)$. Chứng minh rằng $d$ chia hết cho $2010$

Sử dụng $P(a)-P(b)\vdots a-b$ ta được : $P(x^3)-P(1)\vdots x^3-1\vdots x^2+x+1$

và $xQ(x^3)-xQ(1)\vdots x^3-1\vdots x^2+x+1$

$\Rightarrow P(1)+xq(1)\vdots x^2+x+1$.Với $x$ đủ lớn thì $VP>VT$ $\Rightarrow P(1)+xQ(1)= 0\Rightarrow P(1)=Q(1)=0$

$\Rightarrow P(x)= (x-1)R(x);Q(x)= (x-1)H(x);$.Dễ chứng minh $R(x),H(x)\in\mathbb{Z}[x]$

$\Rightarrow P(2011)= 2010R(x);Q(2011)= 2010H(x);$

$\Rightarrow d\vdots 2010$




#296808 Tính $\widehat{BAC}$

Đã gửi bởi Secrets In Inequalities VP on 27-01-2012 - 14:47 trong Hình học

Để mình chém bài này! Mọi nguoi tu vẽ hình nhá!

Gọi E là trung điểm của AB .Suy ra:$ \widehat{AHE}= \widehat{BAH}$ và $ ME//AC$
Vì $ ME//AC$ nên $\widehat{AME}=\widehat{MAC}$
Mà $\widehat{BAH}=\widehat{MAC}$ nên $\widehat{AHE}=\widehat{AME}$
$ \Rightarrow$ AEHM nội tiếp $ \Rightarrow \widehat{AEM}= \widehat{AHM}= 90^{\circ}\Rightarrow \widehat{BAC}= 90^{\circ}$



#367571 $\frac{a^2}{3a^2+(b+c)^2}+\frac{b^2...

Đã gửi bởi Secrets In Inequalities VP on 06-11-2012 - 21:30 trong Bất đẳng thức - Cực trị

Bài 1:
Cho $a,b,c\epsilon \mathbb{R},\geq 0$. CM:
$\frac{a^2}{3a^2+(b+c)^2}+\frac{b^2}{3b^2+(a+c)^2}+\frac{c^2}{3c^2+(a+b)^2}\leq \frac{1}{2}$

http://diendantoanho...c2fracb23b2ac2/



#332762 $a,b,c,x,y,z$ > 0, $x + y + z = 1$ CMR: $ax + by...

Đã gửi bởi Secrets In Inequalities VP on 07-07-2012 - 08:25 trong Bất đẳng thức và cực trị

Cho $a,b,c,x,y,z$ là các số thực dương thỏa mãn : $x + y + z = 1$
CMR: $ax + by + cz + 2\sqrt{(ab + bc + ca)(xy + yz + zx)} \leq a + b + c$

Dùng Cauchy-Schwarz :
$VT\leq \sqrt{(a^{2}+b^{2}+c^{2})(x^{2}+y^{2}+z^{2})}+ \sqrt{2(ab + bc + ca)2(xy + yz + zx)}$
$\leq\sqrt{(a^{2}+b^{2}+c^{2}+2ab+2bc+2ca)(x^{2}+y^{2}+z^{2}+2xy+2yz+2zx)}$
$= \sqrt{(a+b+c)^{2}(x+y+z)^{2}}= a+b+c$



#317708 Cho 3 số dương x,y,z thỏa $x+y+z=6$. CMR $x^2 +y^2 +z^2-xy-yz-...

Đã gửi bởi Secrets In Inequalities VP on 18-05-2012 - 21:09 trong Bất đẳng thức và cực trị

Cho 3 số dương x,y,z thỏa $x+y+z=6$. CMR
$x^2 +y^2 +z^2-xy-yz-zx+xyz\geq 8$

Đặt $ x+y+z=p$ , $ xy+yz+zx=q$ , $\ xyz=r$
$ BDT\Leftrightarrow p^{2}-3q+r\geq 8$ $ \Leftrightarrow 36-3q+r\geq 8\Leftrightarrow 28-3q+r\geq 0$
Ta sẽ CM :$28-3q+r\geq 0$ (1)
Tù BĐT quen thuộc : $ (x+y-z)(y+z-x)(z+x-y)\leq xyz$
Ta rút ra đc : $ r\geq \frac{4pq-p^{3}}{9}$ $ \Rightarrow r\geq \frac{8}{3}q+24$
Suy ra : $ VT\geq 28-3q+\frac{8}{3}q-24= 4-\frac{q}{3}$
Ta có BĐT quen thuộc : $ xy+yz+zx\leq \frac{(x+y+z)^{2}}{3}= 12\Rightarrow q\leq 12$
Do đó : $ VT\geq 4-\frac{q}{3}\geq 4-\frac{12}{3}= 0$
Suy ra : (1) đúng $\ \Rightarrow BDT$ đúng.
Dấu"=" xảy ra khi và chỉ khi $ x=y=z=2$ .




#310442 $2\sum a^{4}+4\sum a^{2}b^{2}\geq 3\sum ab(a^{2}+b^{...

Đã gửi bởi Secrets In Inequalities VP on 14-04-2012 - 22:20 trong Bất đẳng thức và cực trị

Một bài vui vuj ! :icon6: !
Cho $a,b,c$ là các số thục ko âm .
CMR : $2(a^{4}+b^{4}+c^{4})+4(a^{2}b^{2}+b^{2}c^{2}+c^{2}a^{2})\geq 3ab(a^{2}+b^{2})+3bc(b^{2}+c^{2})+3ca(c^{2}+a^{2})$.



#307966 $ \sum \frac{(a+b)^{2}}{a^{2}+b^{2}+2c^{2}}$

Đã gửi bởi Secrets In Inequalities VP on 03-04-2012 - 18:08 trong Bất đẳng thức và cực trị

Cho $ a,b,c> 0$ .CMR:
$ \frac{(a+b)^{2}}{a^{2}+b^{2}+2c^{2}}+\frac{(b+c)^{2}}{b^{2}+c^{2}+2a^{2}}+\frac{(c+a)^{2}}{c^{2}+a^{2}+2b^{2}}$$ \leq 3$



#343701 Tìm Max của Q = 2(a + b + c) - abc

Đã gửi bởi Secrets In Inequalities VP on 05-08-2012 - 17:35 trong Bất đẳng thức và cực trị

Cho các số a, b, c thỏa mãn $a^2+b^2+c^2=9$. Tìm Max của Q = 2(a + b + c) - abc

Here : http://diendantoanho...ight-leq-10abc/
______________
@BlackSelena : xin phép được Close topic :).



#352370 CMR: $2(a+b+c)-abc\leqslant 10$

Đã gửi bởi Secrets In Inequalities VP on 05-09-2012 - 20:14 trong Bất đẳng thức và cực trị

Đây http://diendantoanho...ight-leq-10abc/



#363928 $\sqrt{a+b^{2}}+\sqrt{b+c^{2...

Đã gửi bởi Secrets In Inequalities VP on 22-10-2012 - 20:56 trong Bất đẳng thức và cực trị

Bài toán 2 ( dễ ) Cho $a,b,c> 0$ và $abc=1$. Chứng minh bất đẳng thức sau:

$\frac{3\sum a^{4}b^{4}}{a^{2}+b^{2}+c^{2}}+\frac{8a^{3}}{\left ( bc+a \right )^{3}}+\frac{8b^{3}}{\left ( ca+b \right )^{3}}+\frac{8c^{3}}{\left ( ab+c \right )^{3}}\geq 6$

Công nhận dễ thật ! :P
Theo AM-GM : $3(a^4b^4+b^4c^4+c^4a^4)\geq (a^2b^2+b^2c^2+c^2a^2)^{2}\geq 3a^2b^2c^2(a^2+b^2+c^2)= 3(a^2+b^2+c^2)$
$\Rightarrow \frac{3 (a^{4}b^{4}+b^{4}c^{4}+c^{4}a^{4})}{a^{2}+b^{2}+c^{2}}\geq 3$
Ta cần CM : $\frac{8a^{3}}{\left ( bc+a \right )^{3}}+\frac{8b^{3}}{\left ( ca+b \right )^{3}}+\frac{8c^{3}}{\left ( ab+c \right )^{3}}\geq 3$
$\Leftrightarrow \frac{1}{\left ( 1+\frac{bc}{a} \right )^{3}}+\frac{1}{\left ( 1+\frac{ca}{b} \right )^{3}}+\frac{1}{\left ( 1+\frac{ab}{c} \right )^{3}}\geq \frac{3}{8}$
Đặt $\frac{bc}{a}= x,\frac{ca}{b}= y,\frac{ab}{c}= z\Rightarrow xyz= 1$
$Q.E.D\Leftrightarrow \frac{1}{\left ( 1+x \right )^{3}}+\frac{1}{\left ( 1+y \right )^{3}}+\frac{1}{\left ( 1+z \right )^{3}}\geq \frac{3}{8}$
Quá quen rồi !



#363919 Chứng minh$(a^2+b+\frac{3}{4})(b^2+a+\frac...

Đã gửi bởi Secrets In Inequalities VP on 22-10-2012 - 20:34 trong Bất đẳng thức và cực trị

Cho các số thực không âm a,b.Chứng minh
$(a^2+b+\frac{3}{4})(b^2+a+\frac{3}{4})\geq (2a+\frac{1}{2})(2b+\frac{1}{2})$.

Theo AM-GM : $VT=[(a^2+\frac{1}{4})+b+\frac{1}{2}][(b^2+\frac{1}{4})+a+\frac{1}{2}]\geq (a+b+\frac{1}{2})^{2}= (a+b)^2+a+b+\frac{1}{4}$
$\geq 4ab+a+b+\frac{1}{4}=(2a+\frac{1}{2})(2b+\frac{1}{2})$



#363599 Số học tọa độ :P

Đã gửi bởi Secrets In Inequalities VP on 21-10-2012 - 13:30 trong Số học

Trong mặt phẳng tọa độ $Oxy$ cho $p$ điểm ${A_k}(k,{r_k})$ . $k=0,1,2,..,p-1(p \in \mathbb{P},p> 3)$ . ${r_k}$ là số dư của phép chia $k^2$ cho $p$ .
CMR : Trong các điểm ${A_k}$ không có 3 điểm nào thẳng hàng , không có 4 điểm nào lập thành hình bình hành .



#535546 Cho tứ giác ABCD

Đã gửi bởi Secrets In Inequalities VP on 30-11-2014 - 15:34 trong Hình học

Gọi $X,Y$ là giao của $EF$ với $AB$ và $CD$, $K$ là giao của $AB$ và $CD$

Ta có : $(KXAB)=(KYDC)=-1$ $=>KX.KM=KA.KB=KD.KC=KY.KN$ . suy ra tứ giác $XYNM$ nội tiếp

$=>IM.IN=IX.IY=IE^2$ ( Do $(EFYX)=-1$. đpcm




#375185 Tìm tất cả các cấp số cộng có vô hạn số hạng sao cho tồn tại số $N$...

Đã gửi bởi Secrets In Inequalities VP on 04-12-2012 - 21:25 trong Số học

Tìm tất cả các cấp số cộng có vô hạn số hạng sao cho tồn tại số $N$ mà với mọi $p>N$, nếu $a_p$ nguyên tố thì $p$ cũng là số nguyên tố